반응형

[미적분] 2018학년도 6월 수능모의평가 수학 가형 30번, 수능 수학 가형 30번



이 문제는 문제의 복잡한(?) 조건들을 이용해 요구하는 값을 구하는 문제이다.


2018학년도 6월 모의평가 수학 가형 30번


실수 \(a\)와 함수 \(f(x)=\ln(x^{4}+1)-c\,(c>0인 상수)\)에 대하여 함수 \(g(x)\)를$$g(x)=\int_{a}^{x}{f(t)dt}$$라 하자. 함수 \(y=g(x)\)의 그래프가 \(x\)축과 만나는 서로 다른 점의 개수가 2가 되도록 하는 모든 \(a\)의 값을 작은 수부터 크기순으로 나열하면 \(\alpha_{1},\,\alpha_{2},\,...,\,\alpha_{m}\)(\(m\)은 자연수)이다. \(a=\alpha_{1}\)일 때, 함수 \(g(x)\)와 상수 \(k\)는 다음 조건을 만족시킨다.


(가) 함수 \(g(x)\)는 \(x=1\)에서 극댓값을 갖는다.

(나) \(\displaystyle\int_{\alpha_{1}}^{\alpha_{m}}{g(x)dx}=k\alpha_{m}\int_{0}^{1}{|f(x)|dx}\) 

 

\(mk\times e^{c}\)의 값을 구하시오. [4점]


풀이: 조건 (가)에 의해 \(g(x)\)는 \(x=1\)에서 극댓값을 가지므로 \(g'(1)=f(1)=\ln2-c=0\)이고 \(c=\ln2\)이다.

\(\displaystyle\int_{0}^{1}{|f(x)|dx}=S\)라 하자.그러면 \(f(x)\)가 우함수이므로 \(\displaystyle\int_{-1}^{1}{f(x)dx}=-2S\)이다. \(f(x)\)의 그래프는 다음과 같고

\(g(x)\)가 \(x\)축과 만나는 서로 다른 점의 개수가 2인 경우는 다음과 같다.

그러면 \(m=4\)이고 \(\alpha_{1},\,\alpha_{2},\,\alpha_{3},\,\alpha_{4}\)는 다음과 같다.

이때 \(\alpha_{1}=-\alpha_{4}\)이다. 조건 (나)에서$$\begin{align*}\int_{\alpha_{1}}^{\alpha_{4}}{g(x)dx}&=\int_{\alpha_{1}}^{\alpha_{4}}{(x)'g(x)dx}\\&=[xg(x)]_{\alpha_{1}}^{\alpha_{4}}-\int_{\alpha_{1}}^{\alpha_{4}}{xf(x)dx}\\&=\alpha_{4}g(\alpha_{4})\\&=\alpha_{4}\int_{\alpha_{1}}^{\alpha_{4}}{f(x)dx}\end{align*}$$이고 \(\displaystyle\int_{\alpha_{1}}^{\alpha_{4}}{f(x)dx}=2S\)이므로$$\alpha_{4}\cdot2S=ka_{4}S$$이고 \(k=2\)이다. 

따라서 \(mke^{c}=4\cdot2\cdot e^{\ln2}=4\cdot2\cdot2=16\)이다. 

 

2018학년도 수능 수학 가형 30번


실수 \(t\)에 대하여 함수 \(f(x)\)를$$f(x)=\begin{cases}1-|x-t|&\,(|x-t|\leq1)\\0&\,(|x-t|>1)\end{cases}$$이라 할 때, 어떤 홀수 \(k\)에 대하여 함수$$g(t)=\int_{k}^{k+8}{f(x)\cos(\pi x)dx}$$가 다음 조건을 만족시킨다.


함수 \(g(t)\)가 \(t=\alpha\)에서 극소이고 \(g(\alpha)<0\)인 모든 \(\alpha\)를 작은 수부터 크기순으로 나열한 것을 \(\alpha_{1},\,\alpha_{2},\,...,\,\alpha_{m}\)(\(m\)은 자연수)라 할 때, \(\displaystyle\sum_{i=1}^{m}{\alpha_{i}}=45\)이다.

 

\(\displaystyle k-\pi^{2}\sum_{i=1}^{m}{g(\alpha_{i})}\)의 값을 구하시오. [4점] 


풀이: 함수 \(y=f(x)\)의 그래프는 다음과 같다.

\(\cos\pi x\)는 홀수 정수에서 극소이므로 \(\alpha\)는 홀수 정수이고 \(g(t)\)는 \(x=k\)에서 \(x=k+8\)까지 함수 \(f(x)\cos\pi x\)를 정적분한 함수이므로 \(\alpha\)로 가능한 홀수는 다음과 같고$$k,\,k+2,\,k+4,\,k+6,\,k+8$$문제의 조건에서 \(\displaystyle\sum_{i=1}^{m}{\alpha_{i}}=45\)이므로$$k+(k+2)+(k+4)+(k+6)+(k+8)=5k+20=45$$이고 \(k=5\)이다. \(f(x)\)는 \(x=t\)에 대해 대칭이고, 홀수 정수 \(t\)에 대해 \(\cos\pi t\)는 \(x=t\)에 대해 대칭이므로 \(g(t)\)는 다음과 같다.$$\begin{align*}g(t)&=2\int_{t-1}^{t}{(x-t+1)\cos\pi xdx}\\&=2\left[\frac{(x-t+1)\sin\pi x}{\pi}\right]_{t-1}^{t}-\frac{2}{\pi}\int_{t-1}^{t}{\sin\pi xdx}\\&=\frac{2}{\pi^{2}}[-\cos\pi x]_{t-1}^{t}\\&=\frac{2(\cos \pi t-\cos\pi(t-1))}{\pi^{2}}\\&=-\frac{4}{\pi^{2}}\end{align*}$$따라서$$k-\pi^{2}\sum_{i=1}^{m}{g(\alpha_{i})}=5-\pi^{2}\cdot5\cdot\left(-\frac{4}{\pi^{2}}\right)=5+20=25$$이다.    

반응형
Posted by skywalker222